Find the values of x and y​

Find The Values Of X And Y

Answers

Answer 1

X= 20 & Y= 70 in right-angled triangle .

What is right angle triangle?

A right-angled triangle is a particular kind of triangle in which one of the angles is 90 degrees. The combined angles of the other two are 90 degrees. Perpendicular and the triangle's base make up the sides that the right angle is formed from. The longest of the three sides, known as the hypotenuse, is the third side.

2X + 140 = 180

2X = 40°

   X = 20°

X + Y = 90°

   Y = 70°

Learn more about right-angled triangle

brainly.com/question/3770177

#SPJ13


Related Questions

hellp foo You are considering different investment strategies to save for your retirement.

Option 1: You invest $25/ month at a rate of 3.25% APR compounded monthly for 30 years.

Option 2: You invest $75/ quarter at a rate of 4.00% APR compounded monthly for 30 years.
Option 3: You invest $1,000 at a rate of 6.25% APR compounded monthly for 30 years.
1) Which option was the least amount invested and what was the investment plan?
2) Which option yielded the highest amount at the end of the 30 years and what was the basis of the plan?
3) What is the difference in the principal invested for the highest and lowest final balances? What is the difference in the interest earned?
4) Is it better to invest more money in the beginning or the end of the 30 years?

Answers

1) Option 3 had the least amount invested. The investment plan was to invest a certain amount in the beginning and let it grow at an interest of 6.25% APR compounded monthly for 30 years.

2) Option 2 yielded the highest amount at the end of 30 years. The plan was to invest $75/quarterly i.e, $25/monthly at an interest of 4.00% APR compounded monthly for 30 years.

3) Difference in principle is $8000 and the difference is interest is $2976.83

4) It is better to invest more at the end of 30 years.

Define interest.

When you borrow money, you must pay interest, and when you lend money, you must charge interest. The most common way to represent interest is as a percentage of a loan's total amount per year. The loan's interest rate is denoted by this proportion.

Solution Explained:

First, let's calculate the amount after adding the compound interest using

[tex]A = P(1 + \frac{r}{n})^{nt}[/tex], where P is principal, r = interest rate, n = number of times interest is compounded monthly, and t = time

After calculation (For Op 1 and 2, the principal is $9000 and 3 is $1000)

Option 1 has $15275.50
Option 2 has $17467
Option 3 has $6489.17

Interest for Option 1 = $6275.50
Interest for Option 2 = $8466
Interest for Option 3 = $5489.17

After this solving the questions will be easy to understand.

To learn more about interest, use the link given
https://brainly.com/question/25793394
#SPJ1

Answer:

contributions:
1 $25/ month at a rate of 3.25% APR
2 $75/ quarter at a rate of 4.00%
3 $1,000 at a rate of 6.25% APR

total interest rate:

1 $6275.50

2 $8466

3 $5489.17

final balances:

1 $15275.50

2 $17467

3 $6489.17

questions 1-4

1: Option 3 had the least amount invested

2: option 2
3: Difference is $8000 – and interest difference in $2976.83

4: it’s better to invest more at the end of 30 years

Step-by-step explanation:

i did the assignment

A boat sails 285 miles south and then 132 miles west. What is the magnitude of the boats resultant vector?

Answers

The magnitude of the boats resultant vector is 314.1 mi

What is a vector?

A vector is a physical quantity that has both magnitude and direction.

What is a resultant vector?

A resultant vector is the sum of two or more vectors.

How to find the boats resultant vector?

Since the boat sails 285 miles south and then 132 miles west, we have that its first direction vector is r = (285 mi)j. Also, its direction vector west is r' = -(132 mi)i

So, the resultant vector R = r + r'

=  (285 mi)j + (132 mi)i

=  (132 mi)i + (285 mi)j

So, the magnitude of the resultant vector is R = √(r² + r'²)

So, substituting thevalues of the variables into the equation, we have

R = √(r² + r'²)

R = √((285 mi)² + (132 mi)²)

R = √(81225 mi² + 17424 mi²)

R = √(98649 mi²)

R = 314.08 mi

R ≅ 314.1 mi

So, the resultant vector is 314.1 mi

Learn more about magnitude of resultant vector here:

https://brainly.com/question/28047791

#SPJ1

A self-employed person who completes work on a project-by-project basis is
known as:
A. an independent contractor.
B. a wage-earning employee.
C. an exempt employee.
OD. a salaried employee.
SUBMIT

Answers

Answer:

A: Independent contractor

Step-by-step explanation:

Am I correct? Did I do this right?

Answers

Answer:

Step-by-step explanation:

Yes, you did it right.

What is the sale price including tax to the nearest cent? $595 refrigerator, 20% discount; 9.25% tax

Answers

The sale price including tax for the refrigerator is $520.

How to calculate the price?

From the information, the following can be deduced:

Refrigerator = $595

Discount = 20%

Tax = 9.25%

The sales price will be:

= Refrigerator price + Tax - Discount

= $595 + (9.25% × $595) - (20% × 650)

= $595 + $55 - $130

= $520

The price is $520.

Learn more about price on:

brainly.com/question/1153322

#SPJ1

What is the y-intercept of the graph shown?

Responses


b=2b


b=−2b


b=−3b

b=3b

Answers

The y-intercept of the graph as shown is 2.

How to find the y-intercept of a line?

The equation of a line can be represented as follows:

y = mx + b

where

m = slopeb = y-intercept

The y-intercept is the point where the graph intersects the y-axis. In other words, it is the value of y when x = 0 .

Therefore, let's trace the value of y when x  = 0 on the graph.

Hence, when x = 0, y = 2

Therefore,

b =  y-intercept = 2

learn more on y-intercept here: https://brainly.com/question/16478615

#SPJ1

Answer:b=2b

b=−2b

b=−3b

b=3b

Step-by-step explanation: because I tought myself i know

Pls help would really mean a lot

Answers

Answer:

Use AAA congurency rule

Step-by-step explanation:

1 angle=52

2 angle=58

3 angle=78

What is the first step needed to solve 4 over 7 multiplied by x minus 5 equals negative 13 ? (1 point)

Group of answer choices

Add 5 to both sides

Multiply both sides by 4

Subtract 13 from both sides

Divide both sides by 7

Answers

Answer:add 5 to both sides

Step-by-step explanation:

Question 13(Multiple Choice Worth 2 points)
(Multi-Step Percent Problems MC)

A gaming system costs $600 and is on sale for 35% off. After the discount, there is a 5% tax. What is the final price of the gaming system?

$409.50
$390.00
$220.50
$210.00

Answers

I believe the answer is 409.50

Given that f(x) = x² 13x + 30 and
g(x)= x-3, find f(x) · g(x) and
express the result as a polynomial in simplest form.

Answers

The simplest polynomial form of  f(x)×g(x) is x³ - 16x² + 69x - 90.

What is a polynomial?An expression that consists of variables, constants, and exponents that are combined using mathematical operations like addition, subtraction, multiplication, and division is referred to as a polynomial (No division operation by a variable).Sums of terms of the form k⋅xⁿ, where k is any number and n is a positive integer, makeup polynomials. For instance, the polynomial 3x+2x-5: A description of polynomials.

So, f(x) × g(x):
Calculate as follows:

x(x² - 13x + 30) - 3(x² - 13x + 30)x³ - 13x² + 30x - 3x² + 39x - 90x³ - 16x² + 30x + 39x - 90x³ - 16x² + 69x - 90

Therefore, the simplest polynomial form of  f(x)×g(x) is x³ - 16x² + 69x - 90.

Know more about polynomials here:

https://brainly.com/question/2833285

#SPJ13

4x (x + 10) + 55 = 180

Answers

x = 5/2 and x = -25/2 are the solutions to the equation 4x (x + 10) + 55 = 180.

What is the solution to the quadratic equation?

Given the equation in the question;

4x (x + 10) + 55 = 180

To solve for x, apply distributive property to eliminate the parenthesis.

4x (x + 10) + 55 = 180

4x(x) + 4x(10) + 55 = 180

4x² + 40x + 55 = 180

4x² + 40x + 55 - 180 = 0

4x² + 40x - 125 = 0

Now, we use the quadratic formula to solve the quadratic equation.

x = (-b±√(b² - 4ac)) / (2a)

From 4x² + 40x - 125 = 0

a = 4b = 40 c = -125

Plug these into the formula and solve for x.

x = (-b±√(b² - 4ac)) / (2a)

x = ( -40 ± √( (40)² - ( 4 × 4 × -125))) / ( 2 × 4 )

x = ( -40 ± √( 1600 - ( -2000) )) / ( 8 )

x = ( -40 ± √( 1600 + 2000 )) / ( 8 )

x = ( -40 ± √( 3600 )) / ( 8 )

x = ( -40 ± 60) / ( 8 )

x = ( -40 - 60) / ( 8 ), ( -40 + 60) / ( 8 )

x = ( -100 )/( 8 ), ( 20 )/( 8 )

x = -25/2, 5/2

Therefore, the solution to the quadratic equation is x = -25/2, 5/2.

Learn more about quadratic equations here: brainly.com/question/1863222

#SPJ1

Write a quadratic function h whose only zero is −13.

Answers

The quadratic function whose only zero is -13 is h(x) = x² + 26x + 169

Quadratic function

From the question, we are to write a quadratic function whole only zero is -13.

Since -13 is the only zero of the function,

Then,

x = -13 twice

This means that (x + 13) is a factor of the function

Since x = -13 twice,

Then,

The quadratic function h is

h(x) = (x + 13)(x + 13)

Expand the quadratic function by distributing

h(x) = x(x + 13) + 13(x + 13)

h(x) = x² + 13x + 13x + 169

h(x) = x² + 26x + 169

Hence, the function is h(x) = x² + 26x + 169

Learn more on Quadratic function here: https://brainly.com/question/25549823

#SPJ1

What is 10,483,190 in word form?​

Answers

Answer:

Ten million, Four hundred eighty-three thousand, one hundred ninety

Step-by-step explanation:

Break it down! into the tens digit, hundreds digit, etc.

Ten million four hundred thousand one hundred ninety

There are 12 red balls and 7 green balls in a
non-transparent black bag. What is the
least number of balls we need to take out to be certain we have taken out the
following?
two green or two red balls

Answers

The least number of balls we require to take out to be sure that we have two green or two red balls is 14.

What is the combination of balls?

In the bag, there are a total number of balls of;

Total number of balls = 12 + 7 = 19 balls

Inside the bag, we have 12 red balls and 7 green balls.

The balls are selected without replacement, and this means that they are removed from the bag, and replaced back.

In theory, we could pick 12 balls, and all 12 being red, while also 13, with 12 red and 1 green e.t.c

Therefore, at least 14 balls are needed to guarantee that there are at least 2 green or two red balls

Read more about Combination selection at; https://brainly.com/question/251701

#SPJ1

Find the square root that is a real number

Answers

The square root of √-36 is 0.

What is square root?A number can be obtained by multiplying the square root of that integer by itself. The word "square root" is represented by the symbol "sqrt"—square root of, end square root. The opposite of squaring an integer is finding its square root. In mathematics, a square root is a factor that, when multiplied by itself, equals the original integer. For instance, the square roots of 9 are both 3 and -3.

So, √-36:
Calculate the square root as follows:

√-36²√0= 0

Therefore, the square root of √-36 is 0.

Know more about square roots here:

https://brainly.com/question/3617398

#SPJ13

The volume V of a fixed amount of a gas varies directly as the temperature T and inversely as the pressure P. Suppose that V= 70 cm^3 when T = 420 kelvin and P = 18 kg/cm^2 . Find the pressure when T = 140 kelvin and V = 60 cm^3.

Answers

In a case where the expression is give as volume V of a fixed amount of a gas varies directly as the temperature T and inversely as the pressure P, then the pressure will be 7kg/cm^2.

How can the pressure be calculated?

V = k T/ P

and V= 70 cm^3

T = 420 kelvin

P = 18 kg/cm^2

where K is constant of proportionality,

70 =( k * 420) / 18

k = (70 * 18) / 420

k= 3

Then we were given that the pressure=?

T = 140 kelvin

V = 60 cm^3

then V = k T/ P

P= kT/V

P= (3*140)/60

=7

Then the pressure is 7kg/cm^2

Read more about expression at:

https://brainly.com/question/723406

#SPJ1

If the shared side between angles 1 and 2 has a measure of 2x+7 and the shared side between angles 3 and 4 has a measure of 7x+3, what is the value of X?

Round your answer to the nearest hundredth.

Answers

Answer:

x = 0.8

Step-by-step explanation:

Both mentioned sides have equal length as opposite sides of a parallelogram.

Set equation and solve for x:

2x + 7 = 7x + 37x - 2x = 7 - 35x = 4x = 4/5x = 0.8


Peter and Parker work for a cleaning crew. Peter can clean a room in 15 minutes less time than it takes Parker to do the same job. Working together,
they can clean two rooms in 110 minutes. Approximately how long does it take Peter to clean a room by himself?

Answers

Peter will take 47.5 minutes to clean the room.

In the question ,

it is given that ,

Peter and Parker are working for a cleaning crew ,

let the time taken by Parker to clean the room is = x minutes .

So ,according to the question ,

time taken by Peter to clean the room is = (x - 15) minutes .

Working together, they clean two rooms in 110 minutes

which is represented  , by the equation  ,

x + x - 15 = 110

2x = 110 + 15

2x = 125

x = 125/2

x = 62.5

Parker takes 62.5 minutes and Peter takes 47.5 minutes .

Therefore , Peter will take 47.5 minutes to clean the room .  

Learn more about Equations here

https://brainly.com/question/18067015

#SPJ1

I need help I’ve been stuck on this problem for like 40 minutes already!!!

Answers

Answer is

✅ 12 + 0.25 * 12
Multiply 0.25 * 12 then add 12 = 15

✅ 12 (1 + 0.25)
distribute multiply this expression
12 + 3 = 15

✅ 12 * 1.25 = 15
Multiply for this answer

❌ 12 * 0.25
nope, equals 3

❌ 12 + 0.25
Nope, equals 12.25


The depth of a local river averages 20 ft, which is represented as |−20|. In January, it measured 6 ft deep, or |−6|, and in July, it was 22 ft, or |−22|. What is the difference between depths in January and July?

Answers

Answer: 2 ft.

Step-by-step explanation:
|-20| = 20, |-22| = 22
22 - 20 = 2

A person deposits $8,000 into a savings account at an interest rate of 6.5%. The interest is compounded quarterly. If no other money is deposited into the account, what will the balance be after 12 years? Round to the nearest cent.

Answers

If a person deposits $8,000 into a savings account at 6.5% interest compounded quarterly for 12 years, the balance (future value) will be $17,342.70.

What is the future value?

The future value is the compounded present value of an investment for a future period.

The future value can be computed using the FV formula, table, or an online finance calculator as below.

N (# of periods) = 48 quarters (12 years x 4)

I/Y (Interest per year) = 6.5%

PV (Present Value) = $8,000

PMT (Periodic Payment) = $0

Results:

Future Value (FV) = $17,342.70

Total Interest = $9,342.70

Thus, the future value of a present investment of $8,000 at 6.5% interest compounded quarterly for 12 years is $17,342.70.

Learn more about future values at https://brainly.com/question/24703884

#SPJ1

IQ is normally distributed with a mean of 100 and a standard deviation of 15. a) Suppose one individual is randomly chosen. Find the probability that this person has an IQ greater than 95. Write your answer in percent form. Round to the nearest tenth of a percent. P (IQ greater than 95) = % b) Suppose one individual is randomly chosen. Find the probability that this person has an IQ less than 125. Write your answer in percent form. Round to the nearest tenth of a percent. P (IQ less than 125) = % c) In a sample of 600 people, how many people would have an IQ less than 110? people d) In a sample of 600 people, how many people would have an IQ greater than 140? people

Answers

63 is the answers because actually nvm

Find the product. Simplify your answer
2x² (4x³+2y¹)
12x³y +6
08x5 +4y6
48x5y6
8x5 +8x²y¹

Answers

Answer: The answer is D bc i did the test before hope you have a great day:]

Step-by-step explanation:

8

x

5

6

x

4

+

12

x

3

f(x)=x² +6
Find an equation for the tangent line to the graph of f(x) = x² +6 at (5,31).
y=

Answers

Answer:

y = x^2 + 6       given equation

y = m x + b        equation of straight line required

dy/dx = 2 x       tangent to given curve

dy/dx = slope = 2 * 5 = 10

Thus our line must be of the form

y = 10 x + b

31 = 10 * 5 + b       at the point required

b = 31 - 50 = -19

Then our final equation becomes

y = 10 x - 19

Write an equation in standard form of the line that passes through
(–9, 4) and has a slope of –2/3.

(Add explanation please )

Also, Write the standard form of the equation y + 4 = – ⅗ (x – 8).

Answers

Answer:

2x + 3y = - 63x + 5y = 4

-------------------------------

Standard form of a lineax + by = c

GivenPoint on the line (-9, 4) and a slope m = - 2/3

Use point- slope form and convert to standardy - y₁ = m(x - x₁), where (x₁, y₁) is the point on the liney - 4 = - 2/3(x - (-9))y - 4 = - 2/3(x + 9)3(y - 4) = - 2(x + 9)3y - 12 = - 2x  - 182x + 3y = 12 - 182x + 3y = - 6

Standard of the line y + 4 = - 3/5(x - 8)y + 4 = - 3/5(x - 8)5(y + 4) = - 3(x - 8)5y + 20 = - 3x + 243x + 5y = 24 - 203x + 5y = 4

Question 2 (1 point) Find the area of a triangle with two sides measuring 6 meters and 8 meters with an included angle of 137 degrees. D. 15.8 m² OB. 12. 7m² Oc 16.4m² A. 14.3 m²​

Answers

Answer:

1). True

2). x = 16.4 m^2

3). A = 24, h = 4

4). A = 18.39

5). 162

Step-by-step explanation:

I took this quiz on K12 and got everything correct! Brainliest welcome! :)

A tutor has 21 students in their tutor group. Within this group,
15 students have attended at least one tutorial. Calculate the
percentage of students in this tutor group who have attended at least
one tutorial.
Give your answer correct to the nearest whole number.

Answers

The percentage of students in the tutor group that have attended at least one tutorial is 71%

How to find the percentage?

The percentage of students who have attended at least a single tutorial in the tutor group can be found by the formula:

= Number of students who attended at least one tutorial / Total number of students in tutor group x 100%

Number of students who attended at least one tutorial = 21 students

Total number of students in tutor group = 15 students

The percentage is therefore:

= 15 / 21 x 100%

= 0.7142857 x 100%

= 71.42%

= 71%

Find out more percentages at https://brainly.com/question/24877689

#SPJ1

I’m stuck on this question I’ve tried this question so many times and I just don’t understand so I need help please

Answers

Answer:

see explanation

Step-by-step explanation:

(a)

parallel line segments are indicated by arrows on the segments pointing in the same direction.

from the diagram EA and DC are parallel lines.

(b)

equal lines are indicated by a dash through the lines.

from the diagram AB and BC are equal.

(c)

perpendicular lines are at right angles (90° ) to each other, usually indicated by a small box between them.

from the diagram AE and CD are perpendicular to DE

What number when multiplied by 1 1/4 has a product of 1

Answers

Answer: 4/5 or 0.8

Step-by-step explanation:

[tex]1\frac{1}{4}[/tex] * [tex]\frac{4}{5}[/tex] = [tex]\frac{1}{1}[/tex] = 1

[tex]1\frac{1}{4}[/tex] is also equal to 1.25.

1.25 * 0.8 = 1

Answer:

4/5

Step-by-step explanation:

because 4 is half of a half two fourths are a half and 4/4 are 1 whole

After raining for 3/4 of an hour, a rain gauge 2/5 of the way full if it continued to rain at the same rate for 15 more minutes what fraction of the rain gauge will be filled

Answers

When it rains at the same rate for 15 more minutes, the fraction of the rain gauge that will be filled is 2/15.

What is a fraction?

A fraction is simply a piece of a whole. The number is represented mathematically as a quotient where the numerator and denominator are split.

From the information, after raining for 3/4 of an hour, a rain gauge 2/5 of the way full. It's important to note that 3/4 is (3 × 1/4).

Therefore, we will divide 2/5 by 3 and this will be:

= 2/5 ÷ 3

= 2/5 × 1/3

= 2/15

This is the value of the rain gauge.

Learn more about fractions on:

brainly.com/question/78672

#SPJ1

Other Questions
a loan with borrow options that can be long term with periodic adjustments or short term with more frequent adjustments to the rate is a(n) mortgage. true or false: there are only two basic types of businesses: service businesses and manufacturing businesses. 20 POINTS PLEASE HELP Which provision did the public Treaty of Velasco include?A) Mexican soldiers had one year to leave Texas.B) Santa Anna agreed to never again fight against Texas.C) Texas would become part of the United States.D) Mexican forces could keep all property seized during the war. A 145-ft tower is located on the side of a mountain that is inclined 32 to the horizontal. A guy wire is to be attached to the top of the tower and anchored at a point 55 ft downhill from the base of the tower. Find the shortest length of wire needed. (Round your answer to the nearest foot.) which of the following black artist was the first to gain public recognition in the united states as a portrait painter? How do humanscontribute to the level ofcarbon dioxide in theatmosphere?A. the growing of bone structuresB. the burning of fossil fuels thisC. during photosynthesis How many more monthly payments are made for a five-year loan than for a two-year loan?. Which of the following statements regarding this painting is true?A) It is a pastoral scene.B) It explores the theme of Manifest Destiny.C) It depicts a scene from history (e.g, a battle) or a scene from classical mythology.D) It depicts important virtues, including patriotism and heroism. Why do you think the library of congress holds two copies of all copyright material? consider a 20 kg sphere with a 10 cm radius. what weight would a scale read if the sphere was totally submerged in water? write the equation of the line perpendicular to y= 3/2x + 1/2 that passes through the point (-2,4) Can someone please help ! Are AIDS the same as STD's? How russias war in ukraine helped the fbi crack one of the biggest cybercrime cases in years?. The image shows a rain forest ecosystem. The energy from plants, or producers, acts as the starting point of energy in the ecosystem. This energy is transferred to other organisms in the food web. In which two ways is the total amount of energy conserved in the ecosystem? 17 Can you determine Lack of Encryption and Security Misconfiguration in an organisation? Calculate the mass of 45.0 L of Cl at87.0 C and 950. mm Hg.Use the Ideal Gas Law formula and here are values for R:8.134 (L* kPa)/(mol *K)0.0821 (atm* L)/(mol * K)62.364 (L * mmHg)/(mol *K) A resistor, an inductor, and a capacitor are connected in series to an ac source. What is the phase angle between the voltages of the inductor and capacitor in this rlc circuit?. what method of handling conflict uses an independent third party and avoids the costs of litigation and unilateral decision making?